(-5+ i)(12- i)(-3).

Answers

Answer 1

Answer:

177 - 51i

Step-by-step explanation:

[tex](-5+ i)(12- i)(-3).[/tex]

[tex](-5+ i)(12- i)[/tex]

Apply the distributive property

[tex]-5X12-5(-i)[/tex]+[tex](-i)X12+(i)[/tex][tex](-i)[/tex]

Simplify

[tex]-60+5i+12i-1i^{2}[/tex]

Reduce the imaginary units using the property [tex]i^{2} =-1[/tex]

[tex]-60+5i+12i-1(-1)[/tex]

Simplify and write in the standard form of [tex]a+bi[/tex]

[tex]-59+17i\\(-59+17i)(-3)\\177-51i[/tex]

Hope it helps u:)

Answer 2

Answer:

[tex]177 - 51i[/tex]

Step-by-step explanation:

Given expression:

[tex](-5+i)(12-i)(-3)[/tex]

Use the FOIL method to multiply the first two parentheses:

[tex]\implies \left(-5 \cdot 12 -5 \cdot -i +i \cdot 12 + i \cdot -i\right)(-3)[/tex]

[tex]\implies \left(-60 +5i +12i -i^2\right)(-3)[/tex]

[tex]\implies \left(-60 +17i -i^2\right)(-3)[/tex]

Multiply:

[tex]\implies -60 \cdot -3 +17i \cdot -3 -i^2 \cdot -3[/tex]

[tex]\implies 180-51i+3i^2[/tex]

Apply the imaginary number rule:  i² = -1

[tex]\implies 180-51i+3(-1)[/tex]

Simplify:

[tex]\implies 180-51i-3[/tex]

[tex]\implies 177-51i[/tex]


Related Questions

what is meant by a type i error? a type i error occurs when the null hypothesis is rejected when it is true. the null hypothesis is not rejected when it is false.

Answers

Type I errors (also known as a "false positive") occur when a test erroneously rejects a true null hypothesis. In other words, the test incorrectly concludes that the observed effect is significant or real when, in fact, it is not.

A type I error is a statistical mistake where the null hypothesis is rejected, despite it being true. This is also known as a false positive. This means that the test concluded that the observed effect was significant when in reality, it was not. Type I errors are more likely to occur when the sample size is small or when too many tests are conducted at once. To avoid type I errors, it is important to use an appropriate sample size and to consider the power of the test before conducting it.

Learn more about  error here

https://brainly.com/question/13370015

#SPJ4

find a recurrence relation for the number of bit strings of length n that contain three consecutive 0s. b) what are the initial conditions? c) how many bit strings of length seven contain three consecutive 0s?

Answers

a) A recurrence relation for the number of bit strings of length n that contain three consecutive 0s :

a_n = a_(n-1) + a_(n-2) + a_(n-3) + 2^(n-3)   for n ≥ 3

b) The initial conditions are:

a_0 = a_1 = a_2 = 0 and a_3 = 1

c) There are 47 bit strings of length seven contain three consecutive 0s

Let s_n be a string of length n that does not have 3 consecutive 0's, and a_n be the number of strings  that contain three consecutive 0's

Consider a string of length n -1 that does not have 3 consecutive 0's, s_(n-1)

If we add 1 to this string, then we get a string s_n.

Consider a string s_(n-2)

If we add 10 at the end, then we get a string s_n

Now consider a string s_(n-3)

If we add 100 at the end we get a string s_n.

Now we got all possible strings s_n: that end in 1 (i.e. the last 3 digits could be 001, 011, 101 and 111),

Those strings that end in 10 (i.e. the last 3 digits could be 010 and 110)

and those strings  that end in 100. There are no other possibilities without having 3 consecutive zeros.

In the third case, there are a_(n-3) possibilities. And, in the fourth case, there are 2^(n-3) possibilities.

Hence the recurrence relation is

So, a_n = a_(n-1) + a_(n-2) + a_(n-3) + 2^(n-3)   for n ≥ 3

The initial conditions are a_0 = a_1 = a_2 = 0 and a_3 = 1

The recurrence gives the sequence of positive integers 0, 0, 0, 1, 3, 8, 20, 47, 107, 238, 520, 1121, 2391, . . . .

Hence there are a_7 = 47 bit strings of length seven that contain

three consecutive 0's.

Therefore, a) the recurrence relation:

a_n = a_(n-1) + a_(n-2) + a_(n-3) + 2^(n-3)   for n ≥ 3

b) Initial conditions: a_0 = a_1 = a_2 = 0 and a_3 = 1

c) there are 47 bit strings of length seven

Learn more about the recurrence relation here:

https://brainly.com/question/16931362

#SPJ4

I need help on this question for part A and B​

Answers

Answer:

PART A

x-intercept: (-7,0)

y-intercept: (0,9)

PART B

Answer is A (Top Left One)

Step-by-step explanation:

I left a desmos explanation on my previous answer

Given f (x) = -3x² - 6x +9, find f (-7)

Answers

Answer:

f(-7) = - 96

---------------------------------

Given function:

f(x) = -3x² - 6x +9

Find f(-7) by plugging in the value of x:

f(-7) = - 3(-7)² - 6(-7) +9f(-7) = - 3(49) + 42 +9f(-7) = - 147 + 51f(-7) = - 96

Is the line perpendicular?
The situation is based on football. One player starts a couple of yards in the endzone while the other starts at the 8-9 yard line. The player in the endzone almost scores when he is tracked down by the guy on the 8-9 yard line. So does the starting position of these players form a perpendicular line?

Answers

Yes, the starting position of these players form a perpendicular line.

What is Trigonometry?

The area of mathematics that deals with particular angles' functions and how to use those functions in calculations. There are six popular trigonometric functions for an angle. Sine (sin), cosine (cos), tangent (tan), cotangent (cot), secant (sec), and cosecant are their respective names and acronyms (csc).

Given:

Using Trigonometry

sin [tex]\theta[/tex] = 33.33/100

sin [tex]\theta[/tex] = 0.3333

[tex]\theta[/tex] = [tex]sin^{-1}[/tex] (0.3333)

[tex]\theta[/tex] = 19.469

cos 19.469 = B/ 100

0.9428 = B/ 100

B= 94.28

tan  [tex]\theta[/tex] = P/B

tan  [tex]\theta[/tex] = 94.28/ 33.33

[tex]\theta[/tex] = 70.5

Using Angle Sum property

< 3= 180 - (70.5 + 19.5)

<3 = 180 - 90

<3 = 90

Hence, they form perpendicular line.

Learn more about Trigonometry here:

https://brainly.com/question/29002217

#SPJ1

10. Which expression is equivalent to t+4+3-2.2t?
A 1.2t+7
B-1.2t+7
5.8t
D 10.2t

Answers

Answer: B

Step-by-step explanation:t+4+3−2.2t

Add 4 and 3 to get 7.

t+7−2.2t

Combine t and −2.2t to get −1.2t.

−1.2t+7

17. Louis is offered an interest rate of 6.35%
for an investment with continuous
compounding. What is his equivalent rate
of interest with simple compounding?
[A] 1.23 or 12.3%
[B] 0.0593 or 5.93%
[C] 0.423 or 42.3%
[D] 0.0656 or 6.56%
[E] 0.0615 or 6.15%

Answers

The rate of interest for simple compounding if Louis is offered an interest rate of 6.35% for an investment with continuous compounding, is 0.0656 or 6.56%, so option D is correct.

What is interest?

When the loan is given to you, then some amount is charged to you for the principal amount and that is called interest.

Given:

Louis is offered an interest rate of 6.35% for an investment with continuous compounding,

Here, P will be the same, the amount will be the same and the time period will be the same then,

The amount in simple compounding = The amount in continuous compounding

[tex]P(1 + r)^t = Pe^{Rt}[/tex]

Here, r is the rate of interest for simple compounding,

[tex]1 + r = e^{0.0635}[/tex]

r = 1.0656 - 1

r = 0.06556

r = 0.0656

r = 6.56%

Thus, the rate of interest for simple compounding is 6.56%.

To know more about interest:

https://brainly.com/question/29222674

#SPJ1

Customers of a certain credit card earn points for using the card. The table below shows the number of points earned for the amount spent.

Answers

Answer:

Step-by-step explanation:

18/2 = 9

36/4 = 9

72 / 8 = 9


each dollar spent appears to earn the same number of points


points / dollars = 9

points = 9 · dollars

dollars = points / 9

dollars = 81/9

dollars = 9

how many additional groups would be required to conduct a 3 x 2 x 3 factorial design compared to a 3 x 2 x 2 design?

Answers

One additional group would be required to conduct a 3 x 2 x 3 factorial design compared to a 3 x 2 x 2 design with independent variable.

A 3 x 2 x 3 factorial design requires three independent variables (x1, x2, and x3) with three levels each, for a total of 27 conditions. A 3 x 2 x 2 design, however, would only require two independent variables (x1 and x2) with two levels each, for a total of 12 conditions. To conduct a 3 x 2 x 3 factorial design, one additional group would be required, compared to the 3 x 2 x 2 design. This additional group would provide data for the additional 15 conditions that the 3 x 2 x 3 design would require.

Learn more about independent variable here

https://brainly.com/question/29430246

#SPJ4

2. Name all angle pairs and what makes
them "special".
8
5
7
6
3
2

Answers

Answer:

1 & 3, 2 & 4, 5 & 7, and 6 & 8 are vertically opposite angles

4 & 8, 3 & 7, 1 & 5, and 2 & 6 are corresponding angles

4 & 6 and 3 & 5 are alternate angles

1 & 2, 2 & 3, 3 & 4, 1 & 4, 5 & 6, 6 & 7, 7 & 8, and 5 & 8 are supplementary angles

Write a function in any form that would match the graph shown below 

Answers

The function that matches the graph is y(x)=(1-x)(x+2)^2

What is function?

a function from a set X to the  set of Y assigns to each other element of X exactly one  element of Y. The set X is the  called the domain of to the  function of  the set Y is called as  the condominium of the functions.

From the graph the curve cross the x -axis at X=1

Therefore the expression is (x-1)

And at (-) x axis at X=-2 it is the turning point.

Therefore the expression is

(X+2)^2

And the point on y axis is (0,4)

Therefore the function can be written as

Y(x)=a(x-1)(x+2)^2

4= -4a≈a=-1

Therefore the function is y(x)

(-1)(x-1)(x+2)^2

= y(x)=(1-x)(x+2)^2

Therefore the function that matches the graph is y(x)=(1-x)(x+2)^2

To know more about function click-

https://brainly.com/question/25638609

#SPJ1

The first barn contained 3 times more hay than the second one. After 20 tons of hay
were removed from the first barn and 20 tons were added to the second barn, the

amount of hay in the second barn was -
of the amount remaining in the first barn.
How many tons of hay were there in each barn?

Answers

Answer:

find the size of unknown angle in the following figure

Step-by-step explanation:

angle PQR=115°angle POR=xref. angle POR =2pQR

the solve by putting the given valu

PLEASE HELP- ALGEBRA 2
PICTURE WILL BE PROVIDED
Graph each system of inequalities. Name the coordinates of the vertices of
the feasible region. Find the maximum and minimum values of the given
function for this region.

Answers

Answer:

Step-by-step explanation:

We need to graph all the given constraints(inequalities). The intersection of the of all the graphed system of inequalities gives the vertices of the feasible region.

The first inequality is .  

This is very simple to graph. It is the region bounded by the horizontal lines,

and .

The next constraint or inequality is

To graph this inequality, we need to graph the corresponding linear equation .  

We plot the intercepts  and .

We then draw a solid line through the points.

Next, we test the origin to determine which half plane to shade.

This statement is true so we shade the lower half plane.

We now graph the inequality,

To graph this inequality, we need to graph the corresponding linear equation .  

We plot the intercepts  and .

We then draw a solid line through the points.

Next, we test the origin to determine which half plane to shade.

This statement is true so we shade the lower half plane.

The feasible region is shown in the above diagram.

The vertices of the feasible region are;

We substitute the vertices into the objective function to obtain;

Therefore the maximum value is  and it occurs at .

The minimum value is  and it occurs a

ANSWER TO QUESTIONS 14 We need to graph all the - 1

The acute angle between the vectors a=i-kj and b=i+jis 60° Calculate the possible values of k

no clue how to reach the answer ​

Answers

Answer:

k = (-55) / 8

k = (-3005) / 8

k = (-255 - sqrt(65025 - 510((-255 + sqrt(65025 - 510((-255 + sqrt(65025 - 510(0.309016^2))) / 2)^2)) / 2)^2)) / 2

k = (-255 - sqrt(65025 - 510((-255 + sqrt(65025 - 510((-255 + sqrt(65025 - 1469.59)))))^2)) / 2)

To find the acute angle between two vectors, we can use the dot product formula:

angle = arccos((a * b) / (||a|| * ||b||))

where a and b are the vectors, * is the dot product, and ||a|| and ||b|| are the magnitudes of the vectors a and b, respectively.

In this case, the dot product of a and b is (i - kj) * (i + j) = i^2 - kj * i + kj * i + kj^2 = 2i - k^2j

The magnitudes of the vectors a and b are ||a|| = sqrt(i^2 + (-kj)^2) = sqrt(1 + k^2) and ||b|| = sqrt(i^2 + j^2) = sqrt(2).

Substituting these values into the formula above, we get:

angle = arccos((2i - k^2j) / (sqrt(1 + k^2) * sqrt(2)))

Since the angle is given to be 60 degrees, we can set this equal to 60 degrees and solve for k:

60 = arccos((2i - k^2j) / (sqrt(1 + k^2) * sqrt(2)))

We can use the inverse cosine function to solve for k:

k = sqrt(1 / (cos(60)^2 - (2i / sqrt(1 + k^2) * sqrt(2))^2))

Since cos(60) = 0.5, we can substitute this value in and solve for k:

k = sqrt(1 / (0.5^2 - (2i / sqrt(1 + k^2) * sqrt(2))^2))

k = sqrt(1 / (0.25 - (2i / sqrt(1 + k^2) * sqrt(2))^2))

k = sqrt(1 / (0.25 - (4i^2 / (1 + k^2) * 2)^2))

k = sqrt(1 / (0.25 - (16 / (1 + k^2))^2))

k = sqrt(1 / (0.25 - 256 / (1 + k^2)^2))

k = sqrt((1 + k^2)^2 / (256 - (1 + k^2)^2))

k = sqrt((1 + k^4) / (256 - 1 - 2k^2 - k^4))

k = sqrt((k^4 + 1) / (255 - 2k^2))

We can then solve for the roots of this equation to find the possible values of k:

k = sqrt((k^4 + 1) / (255 - 2k^2))

k^4 - (255 - 2k^2)k^2 + 1 = 0

This is a quartic equation and can be solved using the quartic formula:

k = sqrt((-b +- sqrt(b^2 - 4ac)) / 2a)

where a, b, and c are the coefficients of the polynomial. In this case, a = 1, b = -(255 - 2k^2), and c = 1.

Substituting these values into the quartic formula, we get:

k = sqrt((-(-(255 - 2k^2)) +- sqrt((-(255 - 2k^2))^2 - 4 * 1 * 1)) / 2 * 1)

k = sqrt((255 - 2k^2 +- sqrt((255 - 2k^2)^2 - 4)) / 2)

k = sqrt((255 - 2k^2 +- sqrt(255^2 - 510k^2 + 4k^4)) / 2)

k = sqrt((255 - 2k^2 +- sqrt(255^2 - 510k^2)) / 2)

k = sqrt((255 - 2k^2 +- sqrt(65025 - 510k^2)) / 2)

Solving for the roots of this equation gives us the possible values of k:

k = (-255 + sqrt(65025 - 510k^2)) / 2

k = (-255 - sqrt(65025 - 510k^2)) / 2

The first equation gives us one possible value of k:

k = (-255 + sqrt(65025 - 510k^2)) / 2

Substituting k = (-255 + sqrt(65025 - 510k^2)) / 2 into the second equation gives us the second possible value of k:

k = (-255 - sqrt(65025 - 510((-255 + sqrt(65025 - 510k^2)) / 2)^2)) / 2

Simplifying this expression gives us the final possible value of k:

k = (-255 - sqrt(65025 - 510((-255 + sqrt(65025 - 510((-255 + sqrt(65025 - 510k^2)) / 2)^2)) / 2)^2)) / 2

Therefore, the possible values of k are:

k = (-255 + sqrt(65025 - 510k^2)) / 2

k = (-255 - sqrt(65025 - 510((-255 + sqrt(65025 - 510k^2)) / 2)^2)) / 2

solve for k in each

To solve for k in the first equation, we can isolate k by moving everything else to the right side of the equation:

k = (-255 + sqrt(65025 - 510k^2)) / 2

2k = -255 + sqrt(65025 - 510k^2)

2k + 255 = sqrt(65025 - 510k^2)

(2k + 255)^2 = 65025 - 510k^2

4k^2 + 1020k + 65025 = 65025 - 510k^2

4k^2 + 1530k + 65025 = 0

This is a quadratic equation, and we can use the quadratic formula to solve for k:

k = (-b +- sqrt(b^2 - 4ac)) / 2a

where a, b, and c are the coefficients of the polynomial. In this case, a = 4, b = 1530, and c = 65025.

Substituting these values into the quadratic formula gives us:

k = (-1530 +- sqrt(1530^2 - 4 * 4 * 65025)) / 2 * 4

k = (-1530 +- sqrt(3080400 - 2601000)) / 8

k = (-1530 +- sqrt(477900)) / 8

k = (-1530 +- sqrt(222725)) / 8

k = (-1530 + 1475) / 8

k = (-55) / 8

k = (-1530 - 1475) / 8

k = (-3005) / 8

Therefore, the solutions to the first equation are:

k = (-55) / 8

k = (-3005) / 8

At a restaurant, the bill comes to $70. If you decide to leave a 11% tip, how much is the tip? Give dollars and cents (like 1.23)

Answers

At a restaurant, the bill comes to $70. How much is an 11% tip?

The tip is $7.70

The sin (theta) = -2/5, and theta lies in quadrant IV. Find the exact values of the sine and cosine of 2 theta.

Answers

[tex]\displaystyle\\Answer:\ sin(2\theta)=-\frac{4\sqrt{21} }{25} ,\ cos(2\theta)=\frac{17}{25}[/tex]

Step-by-step explanation:

[tex]\displaystyle\\sin(\theta)=-\frac{2}{5} \ \ \ \ \ \ \ \ 270^0 < \theta < 360^0\\\\sin^2(\theta)+cos^2(\theta)=1\\\\cos^2(\theta)=1-sin^2(\theta)\\\\Hence,\\\\cos^2(\theta)=1-(-\frac{2}{5})^2 \\cos^2(\theta)=1-\frac{4}{25} \\\\cos^2(\theta)=\frac{25(1)-4}{25} \\\\cos^2(\theta)=\frac{21}{25} \\\\[/tex]

Extract the square root of both parts of the equation:

[tex]\displaystyle\\cos(\theta)=б\sqrt{\frac{21}{25} } \\\\cos(\theta)=б\frac{\sqrt{21} }{5} \\\\270^0 < \theta < 360^0\\\\Hence,\\\\cos(\theta)=\frac{\sqrt{21} }{5}[/tex]

[tex]\displaystyle\\a)\ sin(2\theta)=2sin(\theta)cos(\theta)\\\\sin(2\theta)=2(-\frac{2}{5})(\frac{\sqrt{21} }{5})\\\\sin(2\theta)=-\frac{4\sqrt{21} }{25}[/tex]

[tex]\displaystyle\\b)\ cos(2\theta)=cos^2(\theta)-sin^2(\theta)\\\\cos(2\theta)=(\frac{\sqrt{21} }{5})^2-(-\frac{2}{5})^2 \\\\cos(2\theta)=\frac{21}{25}-\frac{4}{25} \\\\cos(2\theta)=\frac{17}{25}[/tex]

When $\dfrac{7}{11}$ is written as a decimal, what is the sum of the first $20$ digits after the decimal point?

Answers

Writing the fraction 7/11 as a decimal gives a repeating decimal 0.636363 the sum of the first two digits is 90

How to add the first twenty digits of a decimal number

The fraction 7/11 is written in fraction to give a repeating deicmal

Where decimal with repeats. Recurring decimal, often known as repeating  decimal, is a decimal number made up only of digits that repeat after the decimal at regular intervals.

The division inform of fraction when converted to decimal gives

0.636363

the sum of the first 20 decimals is 90

this is 6 ten times and 3 ten times

6 * 10 + 3 * 10

60 + 30

90

the sum is 20

Learn more about repeating decimals at:

https://brainly.com/question/16727802
#SPJ1

Choose the equation of the graphed function.

Answers

Answer:

h

Step-by-step explanation:

because it shifts the graph to the left by 6 units

Quadrilateral ABCD is rotated 90° clockwise to produce A'B'C'D' Is each statement true?
AB=A'B'
If AC BD, then A'C' B'D'.
m2ABC Yes No
000

PLEASE HELPP

Answers

Rotation of a shape is a type of transformation. The required answers are:

1. AB=A'B'  (YES)

2.  AC II BD, then A'C' II B'D'    (YES)

3. m<ABC < m<A'B'C'   (NO)

A quadrilateral is a family of shapes which have four sides. Examples are square, rectangle, rhombus etc.

Transformation is a process which can be used to change the orientation or dimensions of a given figure or shape called an object. Some types of transformation are: rotation, translation, reflection, and dilation.

Rotation implies turning an object at a certain angle in a specific direction. The direction could be either clockwise or counterclockwise.

NB: Rotating an object changes only its orientation, but not the length of its dimensions or angles.

So that the required answers to the questions are:

1. AB=A'B'  (YES)

2.  AC II BD, then A'C' II B'D'    (YES)

3. m<ABC < m<A'B'C'   (NO)

Learn more about rotation as a type of translation of objects at https://brainly.com/question/28134390

#SPJ1

What is the value of y when x = 3 in the equation y = 6x - (9 + 5)

Answers

Answer:

y = 4

Step-by-step explanation:

substitute x = 3 into the equation

y = 6(3) - (9 + 5) = 18 - 14 = 4

Answer: y=4

Step-by-step explanation:

y=6(3)-(9+5)

y=18-14

y=4

The rectangular floor of a classroom is 30 feet in length and 35 feet in width. A scale drawing of the floor has a length of 6 inches. What is the perimeter, in inches, of the floor in the scale drawing?

HURRY PLEASE!!!!!!!!!!!!!!!!!!

Answers

Answer:

7 feet

Step-by-step explanation:

Because they divided the length by 5 to get the scaled length so you would do the same for the width.

Answer:

26 inches squared

Step-by-step explanation:

If the real length is 30 feet and the scale drawing is 6 inches its 5:1

So if the actual width is 35 feet the scale drawing will be 7 inches

The perimeter is the sum of all sides of the rectangle, 2 sides have 6 inches and the other 2 sides are 7 inches

6+6+7+7= 26

a company has the following information regarding its forecast performance in the past three periods. icture what is the mean absolute deviation (mad)? question 26 options: 225 -66.7 1200 200

Answers

The mean absolute deviation over three period of time is 200

The absolute value of error in period 1 = 300

The absolute value of error in period 2 = 200

The absolute value of error in period 3 = -100

Total absolute value of error = The absolute value of error in period 1 + The absolute value of error in period 2 + The absolute value of error in period 3

Substitute values in the equation

Total absolute value of error  = 300 + 200 + 100

= 600

The mean absolute deviation = Total absolute value of error / 3

Substitute the values in the equation

The mean absolute deviation  = 600 / 3

= 200

Therefore, the mean absolute deviation is 200

Learn more about mean absolute deviation here

brainly.com/question/10528201

#SPJ4

Lena's phone is 15 cm long and 8 cm wide. Her tablet is 30 cm long and 22 cm wide.
How much additional area is available on her tablet than on her phone?

Answers

Step-by-step explanation:

subtract the area of the phone from that of the phone

Which of these graphs represents a function?

A graph plots an ellipse through the points (negative 5, 3), (negative 5, 2), (0, 1), (1, 1), (2, 1), (5, 3), (2, 4), (0 4) and (negative 2, 4) on the x y coordinate plane.
W. A parabola declines through (negative 5, 2), (negative 3, 0), (negative 2, 0), (0, 1), (1, 2), (2, 3), (3, 5) on the x y coordinate plane.
X.
A curve declines from (0, 6) through (2, 5), (3, 3), (3, 2), (3, 1), (2 points 5, negative 3), (2 points 3, negative 4) and (2 points 2, negative 5) on the x y coordinate plane.
Y. A curve declines from (5, 5), (3, 5), (2, 4 points 9), (negative 1, 4), (negative 2, 4), (negative 3, 3), (negative 2, 2), (0, 1 point 5), (2, 1), (3, 10, (4, 1) and (5, 1) on the x y coordinate plane.
Z.

Answers

The graph of the parabola declines through (-5, 2), (-3, 0), (-2, 0), (0, 1), (1, 2), (2, 3), (3, 5) on the x y coordinate plane represents the function.

What is a Function?

A function is defined as the relation between the x and y coordinates such that no value of x maps to two values of y.

A vertical line test is a test used to determine whether the graph given is a function or not. This is a test where we draw a vertical line passing through a point x. If the graph is a function, then the vertical line will not intersects the graph at two points.

That is, no x coordinate maps to more than 1 y coordinate.

In the first graph of ellipse, the value -5 maps to two values 3 and 5. So this is not a function.

In the parabola all x values only maps to one y value. So this graph represents a function.

In the third graph, the value 3 maps to three values of y, 3, 2, and 1. So this is also not a function.

In the fourth graph, -2 maps to both 4 and 2, which also indicates that the given graph is not a function.

Hence the second graph of the parabola represents a function.

To learn more about Functions, click :

https://brainly.com/question/29300228

#SPJ1

Answer: B. X

Step-by-step explanation; I JUST DID IT(;

Please help me thank you

Answers

Step-by-step explanation:

3.1×10⁵ ÷ 5.4×10⁵

= (3.1/5.4) × 10^5-5

=0.5741 × 10⁰

= 5.7 × 10^-1

The following scores represent students’ test grades in Mr. Preisser's fashion class.

Test Scores

798883937988839379888393798883937988

What is the median score for Mr. Preisser's fashion class?

Answers

Answer:

Median: 85.5

Step-by-step explanation:

Knowledge Needed

Median is the middlemost value of the data set.

For example, we have a data set:

1, 7, 5

Line up in least to greatest.

1, 5, 7

Median is 5.

If there is an even amount of terms, the value of the median is the average of the 2 middlemost terms. For example:

1, 7, 16, 8

Line up in least to greatest.

1, 7, 8 , 16

(7 + 8)/2, average: 7.5

Median is 7.5

Question

Test Scores:

79 88 83 93 79 88 83 93 79 88 83 93 79 88 83 93 79 88

Line them up in least to greatest.

79 79 79 79 79 83 83 83 83 88 88 88 88 88 93 93 93 93

Middle of the set: 83 & 88 (Because there is an even amount of terms).

83 + 88 = 171

171/ 2= 85.5

Median: 85.5

I need help with particular type of psat math question . I'm really struggling pls help me. ​

Answers

Answer:

D. 80 trees

-------------------------------------

For each tree you need 3 + 12 = 15 feet space.

On one side of the street you can plant:

600/15 = 40 trees

Same number is applicable for the opposite side, so the total number is:

40*2 = 80 trees

The matching choice is D.

Please help me with this

Answers

The distance the ball drops in the next 8 second is: 640 m. Using the concept of equation of motion.

What is equation of motion?

An item is considered to be at rest when its position doesn't alter throughout time. An item is considered to be in motion if, over time, its location changes.

It is possible to build a relationship using a series of equations between the body's velocity, acceleration, and the distance it travels in a given amount of time when the body is travelling in a straight line with uniform acceleration. The term "motion equations" refers to these equations.

D = kt²  where D is the constant of proportionality.

D = 80 m

t = 4 seconds

Putting the values we get -

80 = k(4)²

or, k = 80 / 16

or, k = 5.

So the equation of motion is D = 5t²

When t = 8+4 = 12 seconds

now, D = 5(12)² =  5×144

or, D =  720 m

So, the distance the ball drops in the next 8 seconds = 720 - 80

= 640 m.

To know more about equation of motion refer to:

https://brainly.com/question/27821888

#SPJ1

Linear Functions: Model from Two Points-Quiz-Level H
Ready
4) A bathtub has some water in it. Mia turns on the faucet to add more water. The total amount of
water in gallons, y, is a function of the time in minutes since Mia turns on the faucet, a.
4) The graph of the linear function passes through the points (4, 24) and (6, 30).
What is the equation of the function?
?

Answers

The equation of function for the given problem is y = 3x + 12.

What is point slope form of the line?

For linear equations, the general form is y - y1 = m(x - x1).

It draws attention to the line's slope and one of the line's points (that is not the y-intercept).

Given:

A bathtub has some water in it. Mia turns on the faucet to add more water.

The total amount of water in gallons, y, is a function of the time in minutes since Mia turns on the faucet, a.

The graph of the linear function passes through the points (4, 24) and

(6, 30).

We have to find the equation of function.

Let the linear function passes through the points (4, 24) and (6, 30).

First to find the slope of equation using given points.

[tex]m = \frac{y_2-y_1}{x_2-x_1} = \frac{30-24}{6-4} = \frac{6}{2} = 3[/tex]

Now to find the equation of function.

Consider the point slope form of the line,

[tex]y-y_1=m(x-x_1)[/tex]

Plug the values of m = 3 and [tex](x_1,, y_1) = (4, 24)[/tex]

[tex]y-24=3(x-4)\\y-24=3x-12\\y=3x-12+24\\y=3x+12[/tex]

Hence, the equation of function for the given problem is y = 3x + 12.

To know more about point slope form of line, click on the link

https://brainly.com/question/29797287

#SPJ1

A supervisor keeps record about her team she worked out the value of sales each salesperson made this morning

Answers

The total value of the sales today is £1581 and the total value of the sales for each salesperson today is:

Mo = £330 +  £247 = £577,

Daz = £180 + £259 = £439,

Colin = £300 + £265 = £565,

What are arithmetic operations?

Arithmetic operations is a branch of mathematics that studies numbers and the operations on numbers that are useful in all other branches of mathematics. It consists primarily of operations like addition, subtraction, multiplication, and division.

We have,

The value of sales each salesperson made this morning from the bar chart:

Mo = £330,  Daz = £180,  Colin = £300,

The value of sales each salesperson made this afternoon from the table given:

Mo = £247,  Daz = £259,  Colin = £265,

so, the total value of the sales today is :

= £330 + £180 + £300 + £247 + £259 + £265

= £1581.

also the total value of the sales for each salesperson today is:

Mo = £330 +  £247 = £577,

Daz = £180 + £259 = £439,

Colin = £300 + £265 = £565,

Hence, the total value of the sales today is £1581 and the total value of the sales for each salesperson today is:

Mo = £330 +  £247 = £577,

Daz = £180 + £259 = £439,

Colin = £300 + £265 = £565,

To learn more about arithmetic operations visit,

https://brainly.com/question/4721701

#SPJ1

Other Questions
at the beginning of the movie our hero prof. jack hall and his colleagues are found on the larsen b ice shelf in antarctica doing what? What is the molarity of 4 g of nacl in 3, 800 ml of solution? molar mass nacl = 58. 44 g/mol. is the amount of mental effort required to understand a message. Interpersonal communication is high involvement. It requires a great deal of thought and action. Thus, health professionals, family members, and friends tend to have high impact researchers have found that mixtures of phospholipids placed in water or salt solutions tend to spontaneously form: the half equivalence point is in the middle of the buffer region. in order to reach the half equivalence point in their titration, veronica needed to add 24.47 ml of koh to 50.00 ml of 0.368 m hf. what is the concentration of conjugate base at the half equivalence point? note: do not use scientific notation or units in your response. sig figs will not be graded in this question, enter your response to four decimal places. carmen may add or remove digits from your response, your submission will still be graded correctly if this happens. cross-industry standard process for data mining (crisp-dm) consists of six phases. of the six, which one represents the phase where data wrangling occurs? A child asks his parents for some money. The parents make the following offers.Fathers offer: The child flips a coin. If the coin lands heads up, the father will give the child $20. If the coin lands tails up, the father will give the child nothing.Mothers offer: The child rolls a 6-sided die. The mother will give the child $3 for each dot on the upside of the die.Which offer has the greater expected value?answer choices:Mother's OfferFather's Offer Lisa's pet shop has 2 fish tanks. Tank A contains smaller fish who are fed 1 gram of food each per day. Tank B contains larger fish who are fed 2 grams of food each per day. If Tank B contains 2/3 the number of fish that Tank A contains, will Lisa ever feed both tanks the same amount of food? you are trying to decide between two vacation options for the summer. the first option is a cruise through the caribbean sea that you have determined would provide you with 98,000 utils of satisfaction and will cost $2,800. the second option is a beach vacation in the florida keys that you have determined would provide you with 74,000 utils of satisfaction and will cost $1,850. assuming both options are affordable, you would choose to take vacation. Need Help!!!!!!!!! Solve for side similar triangle if the minimum expected regret (i.e., minimum eol) is computed, it indicates to a decision-maker the: group of answer choices the expected value of perfect information (evpi) highest expected monetary value (emv) expected payoff under certainty expected profit with perfect information (eppi) bartley are the plates in contact at point a (where the plate appears dark) or at point b (where the plate appears bright)? explain your reasoning. FILL IN THE BLANK. ________ are the deeply held beliefs that guide behavior and tell members of an organization how to perceive and think about things. 3) The human body can do amazing things. However, supplementation is essential to get the most out of it. Only NatureMan can increase your muscle size and strength in just weeks. Nothing is more important than ensuring the correct ingestion of vitamins, minerals, and proteins. The new improved Nature Man vitamin will ensure you have all that you need. Additionally, Nature Man has many additives that will boost your performance such as IGF colostrums, nucleotides, L-arginine, creatine and lipotropics. Be all nature intended you to be with Nature Man vitamins! 5. List four questions you have about this product: a. b. c. d. 6. List two reasons you should be wary of using this product. a. b. how did the supreme court extend freedom of speech to protect against acts of state governments in 1925? Throughout the reflection, make sureyou have a copy of the Student Guide and your data tables.In this experiment, thewas intentionally manipulated. This was the independent variableThe dependent variable measured was the people with depression show different sleep patterns, a major one being a striking reduction of time spent in ____ sleep. sales management consists of three interrelated functions: , sales plan implementation, and salesforce evaluation. multiple choice sales plan formulation salesforce size determination salesforce identification salesforce communication salesforce training 1. Part A: Graph quadrilateral WXYZwith vertices W(-4,5), X(3,5), Y(3,-6), andZ(-4,-6).*plot this quadrilateral Part B: Find the length of the followingsidesWX=XY=YZ=WZ= a television camera is positioned 4,000 ft from the base of a rocket launching pad. the angle of elevation of the camera has to change at the correct rate in order to keep the rocket in sight. also, the mechanism for focusing the camera has to take into account the increasing distance from the camera to the rising rocket. let's assume the rocket rises vertically and its speed is 900 ft/s when it has risen 3,000 ft. (Round your answers to three decimal places.) (a) How fast is the distance from the television camera to the rocket changing at that moment? 480 ft/s (b) If the television camera is always kept aimed at the rocket, how fast is the camera's angle of elevation changing at that same moment? 0.002 rad/s